Så i $ {\ bf R} ^ {n \ gånger p} $ har vi Frobenius inre produkt ges av $$ \ langle A, B \ rangle = \ text {tr} (A ^ TB) $$

som kan tolkas som den euklidiska inre produkten på $ {\ bf R} ^ {np } $. Min förståelse är att alla inre produkter på $ {\ bf R} ^ {np} $ kan skrivas som $$ a ^ TPb $$ för $ P $ positivt-definitivt. Det bästa jag kunde göra för att försöka förlänga Frobenius inre produkt på $ {\ bf R} ^ {n \ gånger p} $ är något av formen $$ \ langle A, B \ rangle = \ sum_ {i = 1} ^ N \ text {tr} ((X_iAY_i) ^ T (X_iBY_i)) $$ för $ X_i \ i {\ bf R} ^ {m_i \ gånger n} $ och $ Y_i \ i {\ bf R} ^ {p \ times q_i} $ all full rang. Men jag skulle vilja veta om detta täcker alla inre produkter på $ {\ bf R} ^ {np} $, eller om det kanske är mer komplicerat än nödvändigt på grund av uppsägningar.

Jag kan hitta motsvarande $ P $ -matris för varje specifik matrisinre produkt genom att ta standardbasen för $ {\ bf R} ^ {n \ gånger p} $ och bilda matrisen

\ begin {bmatrix} \ langle E_1 , E_1 \ rangle & \ langle E_1, E_2 \ rangle & \ prickar & \ langle E_1, E_ {np} \ rangle \\ \ langle E_2, E_1 \ rangle & \ langle E_2, E_2 \ rangle & & \ vdots \\ \ vdots & & \ ddots \\ \ langle E_ {np }, E_1 \ rangle & \ dots & \ dots & \ langle E_ {np }, E_ {np} \ rangle \ end {bmatrix}

men jag vet inte om den allmänna formen för en matrisinreprodukt jag gav ovan täcker alla positivt bestämda matriser $ P $.

Uppdatering:

nyare version av den här frågan den MathOverflow: https://mathoverflow.net/questions/229675/extending-the-trace-inner-product-to-all-matrix-real-inner-products

Kommentarer

  • Välkommen till SciComp.SE! Det här är en intressant fråga, men verkar mycket mer lämplig för math.stackexchange.com . (Om det inte finns ' en koppling till ett beräkningsvetenskapligt problem saknar jag ', i vilket fall det ' Det är bra om du kan lägga till det.)
  • @ChristianClason, det ' är relaterat till optimering på matrisgrenrör med Riemannian-mått, sedan Riemannian mätvärden är inre produkter i tangentutrymmet. Det ' är nästan säkert för avancerat för Math.SE, den enda andra lämpliga platsen skulle vara MathOverflow. Jag kanske faktiskt har hittat vad jag tycker är en lösning som jag kan lägga upp som ett svar när jag gör det röriga arbetet med att bevisa att det är en lösning, men om du ' vill migrera detta till MathOverflow I ' m ok med det. Jag ' Jag lägger till optimeringskontexten när jag får en chans.
  • Matrisen $ P $ måste också vara symmetrisk, inte bara positiv.
  • @WolfgangBangerth, positivt-definitivt förstås innebära symmetrisk.
  • Inte för alla författare innebär positiv bestämning symmetri.

Svar

Du kan se en inre produkt som en operation $ f (a, b) = \ left < a, b \ right > $, dvs det är en bilinear funktion som (i) returnerar ett icke-negativt tal, (ii) uppfyller förhållandet $ f (a, b) = f (b, a) $.

För vektorer $ a, b \ in \ mathbb R ^ n $ kan alla tvålinjära funktioner som uppfyller dessa egenskaper skrivas som $$ f (a, b) = \ sum_ {i, j = 1 } ^ n a_i P_ {ij} b_j $$ där $ P $ är symmetrisk och positiv bestämd. För matriser $ a, b \ i \ mathbb R ^ {n \ gånger p} $ kan alla sådana funktioner skrivas som $$ f (a, b) = \ sum_ {i, k = 1} ^ n \ sum_ { j, l = 1} ^ p a_ {ij} P_ {ijkl} b_ {kl} $$ där nu $ P $ är en tensor av rang 4 som är symmetrisk i den meningen att $ P_ {ijkl} = P_ {klij} $ och positivt definitivt i den meningen att $ f (a, a) > 0 $ för alla $ a \ neq 0 $.

Din fråga kokar ner till om varje $ P $ som uppfyller sådana villkor kan skrivas ett formulär som härrör från vektorerna $ X_i, Y_i $. Jag tror att svaret på detta är nej. Detta är helt enkelt så för att (för enkelhetens antagande $ n = p $) symmetrisk $ P $ har (asymptotiskt) $ n ^ 4/2 $ frihetsgrader, medan $ n $ vektorerna $ X_i, Y_i $ bara har $ 2n ^ 2 $ frihetsgrader. Med andra ord tror jag inte att för tillräckligt stora $ n $ har ditt tillvägagångssätt tillräckligt många frihetsgrader.

Kommentarer

  • I tror faktiskt att svaret är ja, jag ' kommer att lägga om den här frågan på matematikflöde med mina uppdaterade resultat.
  • Ja ditt argument att antalet parametrar växer quarticly i vektorn inre produktutrymme medan endast kvadratiskt i matrisen är inre produktutrymme övertygande, men eftersom utrymmet i slutändan är ändligt bör vi kunna övervinna detta genom att öka $ N $ på lämpligt sätt.
  • Jag ber om ursäkt för att jag publicerade en nyare version av denna fråga på MathOverflow, men den ' är tillräckligt uppdaterad Jag tyckte att det var lämpligt, här är länken om du vill för att överföra ditt svar dit eller uppdatera ditt svar baserat på den nyare versionen. mathoverflow.net/questions/229675/…
  • @Thoth Observera att @ ChristianClason rådde att du skickar din fråga på math.stackexchange.com, inte på mathoverflow.net. Det är två olika sajter med olika syften och målgrupper.
  • @FedericoPoloni ja jag vet, och om du läser vad jag skrev sa jag till honom att jag tyckte att det var för avancerat för Math.SE och det är osannolikt att det skulle bli ett svar där.

Lämna ett svar

Din e-postadress kommer inte publiceras. Obligatoriska fält är märkta *